Difference between revisions of "1988 AHSME Problems/Problem 30"

m (See also)
(Problem)
Line 6: Line 6:
 
<math>\textbf{(A)}\ \text{0}\qquad
 
<math>\textbf{(A)}\ \text{0}\qquad
 
\textbf{(B)}\ \text{1 or 2}\qquad
 
\textbf{(B)}\ \text{1 or 2}\qquad
\textbf{(C)}\ \text{3, 4, 5 or 6}\qquad\\
+
\textbf{(C)}\ \text{3, 4, 5 or 6}\qquad
\textbf{(D)}\ \text{more than 6 but finitely many}\qquad\\
+
\textbf{(D)}\ \text{more than 6 but finitely many}\qquad
\textbf{(E) }\infty</math>  
+
\textbf{(E) }\infty</math>
 
 
  
 
==Solution==
 
==Solution==

Revision as of 05:56, 14 April 2016

Problem

Let $f(x) = 4x - x^{2}$. Give $x_{0}$, consider the sequence defined by $x_{n} = f(x_{n-1})$ for all $n \ge 1$. For how many real numbers $x_{0}$ will the sequence $x_{0}, x_{1}, x_{2}, \ldots$ take on only a finite number of different values?

$\textbf{(A)}\ \text{0}\qquad \textbf{(B)}\ \text{1 or 2}\qquad \textbf{(C)}\ \text{3, 4, 5 or 6}\qquad \textbf{(D)}\ \text{more than 6 but finitely many}\qquad \textbf{(E) }\infty$

Solution

See also

1988 AHSME (ProblemsAnswer KeyResources)
Preceded by
Problem 29
Followed by
Last Question
1 2 3 4 5 6 7 8 9 10 11 12 13 14 15 16 17 18 19 20 21 22 23 24 25 26 27 28 29 30
All AHSME Problems and Solutions

The problems on this page are copyrighted by the Mathematical Association of America's American Mathematics Competitions. AMC logo.png